NURS 302 Final

Ace your homework & exams now with Quizwiz!

A client who has been taking fluvoxamine (Luvox) without significant improvement asks a nurse, "I heard about something called a monoamine oxidase inhibitor (MAOI). Can't my doctor add that to my medications?" Which is an appropriate nursing reply?

"A combination of an MAOI and Luvox can lead to a life-threatening hypertensive crisis." Rationale: The nurse should explain to the client that combining an MAOI and Luvox can lead to a life-threatening hypertensive crisis. Symptoms of hypertensive crisis include severe occipital and/or temporal pounding headaches with occasional photophobia, sensations of choking, palpitations, and a feeling of "dread."

A nursing instructor is teaching students about cirrhosis of the liver. Which of the following student statements about the complications of hepatic encephalopathy should indicate that further student teaching is needed? Select all that apply.

"A diet rich in protein will promote hepatic healing." "In this condition, blood accumulates in the abdominal cavity." Rationale: The nursing instructor should understand that further teaching is needed if the nursing student states that a diet rich in protein will promote hepatic healing or that this condition causes blood to accumulate in the abdominal cavity (ascites), because these are incorrect statements. The treatment of hepatic encephalopathy requires abstention from alcohol, temporary elimination of protein from the diet, and reduction of intestinal ammonia by means of neomycin or lactulose. This condition occurs in response to the inability of the liver to convert ammonia to urea for excretion.

A highly agitated client paces the unit and states, "I could buy and sell this place." The client's mood fluctuates from fits of laughter to outbursts of anger. Which is the most accurate documentation of this client's behavior?

"Agitated and pacing. Exhibiting grandiosity. Mood labile." Rationale: The nurse should document that this client's behavior is "Agitated and pacing. Exhibiting grandiosity. Mood labile." The client is exhibiting signs of irritation accompanied by aggressive behavior. Grandiosity refers to an exaggerated sense of power, importance, knowledge, or identity.

A family member is seeking advice about an elderly parent who seems to worry unnecessarily about everything. The family member states, "Should I seek psychiatric help for my mother?" Which is an appropriate nursing reply?

"Anxiety is considered abnormal when it is out of proportion to the stimulus causing it and when it impairs functioning." Rationale: The most appropriate reply by the nurse is to explain to the family member that anxiety is considered abnormal when it is out of proportion and impairs functioning. Anxiety is a normal reaction to a realistic danger or threat to biological integrity or self-concept.

A nursing instructor is teaching about the etiology of dissociative disorders from a psychoanalytical perspective. What student statement about clients diagnosed with this disorder indicates that learning has occurred?

"Dissociative behaviors occur when individuals repress distressing mental information from their conscious awareness."

On the fifth day postpartum, a woman calls her healthcare provider and reports pronounced fatigue, sadness and tearfulness. She states, "I feel so overwhelmed, I don't know what to do!" Which of the following questions is most appropriate for the healthcare provider to ask?

"Do you ever think about harming yourself or your baby?" Rationale: Feelings of fatigue, sadness, and tearfulness can be common symptoms experienced in the postpartum period. The healthcare provider will want to ask questions that will help distinguish postpartum blues from postpartum depression. Whereas patients who are diagnosed with postpartum depression may experience thoughts of harming themselves or the infant, this is not a finding in postpartum blues.

A patient diagnosed with bipolar disorder is prescribed lithium carbonate (Lithobid). When teaching the patient about the medication, which of these statements is a priority for the healthcare provider to include?

"Drink lots of fluids, especially if you are active during hot weather." Rationale: Lithium is an inorganic ion similar to other ions such as potassium and sodium. If sodium levels are low, the kidneys will retain lithium, which could result in toxicity. Lithium increases urine output and antagonizes the effects of antidiuretic hormone. In order to avoid dehydration, patients should be instructed to drink 10 - 12 glasses of water each day. Additional fluids will be needed during strenuous activity, in hot weather, or if the patient experiences fluid loss through vomiting or diarrhea.

What tool should a nurse use to differentiate occasional spontaneous behaviors of children from behaviors associated with bipolar disorder?

"FIND" tool Rationale: The Consensus Group recommends that clinicians use the FIND tool to differentiate occasional spontaneous behaviors of children from behaviors associated with bipolar disorder. FIND is an acronym that stands for frequency, intensity, number, and duration and is used to assess behaviors in children.

Parents ask a nurse how they should reply when their child, diagnosed with schizophrenia, tells them that voices command him to harm others. Which is the appropriate nursing reply?

"Focus on the feelings generated by the hallucinations and present reality."

Parents ask a nurse how they should reply when their child, diagnosed with schizophrenia, tells them that voices command him to harm others. Which is the appropriate nursing reply?

"Focus on the feelings generated by the hallucinations and present reality." Rationale: The most appropriate response by the nurse is to instruct the parents to focus on the feelings generated by the hallucinations and present reality. The parents should maintain an attitude of acceptance to encourage communication but should not reinforce the hallucinations by exploring details of content. It is inappropriate to present logical arguments to persuade the client to accept the hallucinations as not real.

The nurse is caring for a client diagnosed with bipolar disorder. On interaction with the client, the nurse suspects that the client is in the delirious mania stage. Which statement of the client supports the nurse's conclusion? Select all that apply

"Go away, leave me alone" "I dont want to go into the bathroom because someone in there and planning to kill me" "I saw a demon standing in front of me"

A client diagnosed with major depressive disorder states, "I've been feeling 'down' for 3 months. Will I ever feel like myself again?" Which reply by the nurse will best assess this client's affective symptoms?

"Help me understand what you mean when you say, 'feeling down'?" Rationale: The nurse is using a clarifying statement in order to gather more details related to this client's mood.

A patient is receiving care after being diagnosed with generalized anxiety disorder (GAD). Which of these statements made by the patient indicate to the healthcare provider that the patient is beginning to show signs of improvement?

"I can tell when I'm beginning to experience anxiety." Rationale: GAD is characterized by excessive worrying that may result in problems such as a hyperarousal, muscle tension, difficulty relaxing, and impaired sleep patterns. Patients diagnosed with GAD often engage in avoidance behaviors. Recognizing when symptoms of anxiety occur is an initial goal for the patient. Once anxiety is recognized, the patient can employ coping skills to manage the anxiety. Mediations can be helpful in managing GAD, but should be used in conjunction with cognitive-behavioral therapies.

A mother with a history of chronic heroin use has lost custody of her children due to abuse and neglect. She has been admitted to an inpatient drug rehabilitation program. Which client statement should a nurse associate with a positive prognosis for this client?

"I cannot control my use of heroin. It's stronger than I am." Rationale: A positive prognosis is more likely when a client admits that he or she is addicted to a substance and has a loss of control. One of the first steps in the 12-step model for treatment is for the client to admit powerlessness over the substance.

Which client statement would demonstrate a common characteristic of Cluster "B" personality disorder?

"I didn't have the money for the ring, so I just took it."

A patient diagnosed with paranoid schizophrenia states, "My roommate is plotting to have others kill me." Which is the appropriate nursing response?

"I find that hard to believe."

A patient diagnosed with paranoid schizophrenia states, "My roommate is plotting to have others kill me." Which is the appropriate nursing response?

"I find that hard to believe." Rationale: This patient is experiencing a persecutory delusion. This nursing response is an example of "voicing doubt," which expresses uncertainty as to the reality of the client's perceptions. This is an appropriate therapeutic communication technique in dealing with clients who are experiencing delusional thinking Option B: Asking why should be avoided because it demands explanation and may put patient in defensive mode. Option C: This is false re-assurance. The nurse does not completely know the other patient. Option D: This option is agreeing with the patient thus strengthening the delusion. This is a form of non-therapeutic communication.

A nurse tells a client that the nursing staff will start alternating weekend shifts. Which response should a nurse identify as characteristic of clients diagnosed with obsessive-compulsive personality disorder?

"You can't make these kinds of changes! Isn't there a rule that governs this decision?"

An isolative client was admitted 4 days ago with a diagnosis of major depressive disorder. Which nursing statement would best motivate this client to attend a therapeutic group being held in the milieu?

"I'll walk with you to the day room. Group is about to start."

An isolative client was admitted 4 days ago with a diagnosis of major depressive disorder. Which nursing statement would best motivate this client to attend a therapeutic group being held in the milieu?

"I'll walk with you to the day room. Group is about to start." Rationale: A client diagnosed with major depressive disorder exhibits little to no motivation and must be actively directed by staff to participate in therapy. It is difficult for a severely depressed client to make decisions, and this function must be temporarily assumed by the staff.

A patient diagnosed with depression is prescribed a monoamine oxidase inhibitor (MAOI). When teaching the patient about the medication, which statement made by the patient indicates the need for additional teaching?

"I'm glad that I can have pepperoni on my pizza." Rationale: Patients taking MAOIs need to adhere to numerous dietary restrictions. The patient will need to avoid consuming foods which are high in tyramine. Processed meats such a pepperoni are high in tyramine. Combining tyramine-rich foods with a monoamine oxidase inhibitor (MAOI) can result in a hypertensive crisis.

A nurse learns at report that a newly admitted client experiencing mania is demonstrating grandiose delusions. The nurse should recognize that which client statement would provide supportive evidence of this symptom?

"I'm the world's most perceptive attorney." Rationale: Grandiosity is defined as a belief that personal abilities are better than anyone else's. This client is experiencing delusions of grandeur, which are commonly experienced in mania.

A patient presents to the clinic with a report of fatigue and difficulty concentrating. Which additional statement made by the patient would alert the healthcare provider to possible marijuana use?

"I've noticed that my eyes are red lately."

A patient presents to the clinic with a report of fatigue and difficulty concentrating. Which additional statement made by the patient would alert the healthcare provider to possible marijuana use?

"I've noticed that my eyes are red lately." Rationale: THC (tetrahydrocannabinol), the active ingredient in marijuana, affects thinking, memory, appetite, and movement. Hallucinogens such as LSD (lysergic acid diethylamide) can cause flashbacks. Marijuana use can cause corneal vasodilation and conjunctivitis. It's more likely that marijuana would increase appetite, decrease anxiety, and promote sleep.

The family of a client diagnosed with conversion disorder asks the nurse, "Will his paralysis ever go away?" Which of these responses by the nurse is evidence-based?

"Most symptoms of conversion disorder resolve within a few weeks."

A nurse is caring for a client who has been prescribed disulfiram (Antabuse) as a deterrent to alcohol relapse. Which information should the nurse include when teaching the client about this medication?

"Reactions to combining Antabuse with alcohol can occur for as long as 2 weeks after stopping the drug." Rationale: If Antabuse is discontinued, it is important for the client to understand that the sensitivity to alcohol may last for as long as 2 weeks.

An elderly client diagnosed with schizophrenia takes an antipsychotic and a beta-adrenergic blocking agent (propranolol) for hypertension. Understanding the combined side effects of these drugs, the nurse would most appropriately make which statement?

"Rise slowly when you change position from lying to sitting or sitting to standing."

An elderly client diagnosed with schizophrenia takes an antipsychotic and a beta-adrenergic blocking agent (propranolol) for hypertension. Understanding the combined side effects of these drugs, the nurse would most appropriately make which statement?

"Rise slowly when you change position from lying to sitting or sitting to standing." Rationale: The most appropriate statement by the nurse is to instruct the client to rise slowly when changing positions. Antipsychotic medications and beta blockers cause a decrease in blood pressure. When given in combination, this side effect places the client at risk for developing orthostatic hypotension.

A client diagnosed with schizophrenia tells a nurse, "The "Shopatouliens" took my shoes out of my room last night." Which is an appropriate charting entry to describe this client's statement?

"The client is expressing a neologism." Rationale: The nurse should describe the client's statement as experiencing a neologism. A neologism is when a client invents a new word that is meaningless to others but may have symbolic meaning to the client. Word salad refers to a group of words that are put together randomly.

A nursing instructor is teaching students about clients diagnosed with histrionic personality disorder and the quality of their relationships. Which student statement indicates that learning has occurred?

"Their interpersonal relationships tend to be shallow and fleeting, serving their dependency needs."

Which statement should indicate to a nurse that an individual is experiencing a delusion?

"There's an alien growing in my liver." Rationale: The nurse should recognize that a client who claims that an alien is inside his or her body is experiencing a delusion. Delusions are false personal beliefs that are inconsistent with the person's intelligence or cultural background.

A nursing instructor is teaching about specific phobias. Which student statement should indicate that learning has occurred?

"These clients have overwhelming symptoms of panic when exposed to the phobic stimulus." Rationale: The nursing instructor should evaluate that learning has occurred when the student knows that clients experiencing phobias have a panic level of fear that is overwhelming and unreasonable. Phobia is fear cued by a specific object or situation in which exposure to the stimulus produces an immediate anxiety response.

A nursing instructor is teaching about the prevalence of bipolar disorder. Which student statement indicates that learning has occurred?

"This disorder is more prevalent in the higher socioeconomic groups." Rationale: The nursing student is accurate when stating that bipolar disorder is more prevalent in higher socioeconomic groups. Theories consider both hereditary and environmental factors in the etiology of bipolar disorder.

A client is questioning the nurse about a newly prescribed medication, acamprosate calcium (Campral). Which is the most appropriate reply by the nurse?

"This medication will help you maintain your abstinence." Rationale:Campral has been approved by the U.S. Food and Drug Administration (FDA) for the maintenance of abstinence from alcohol in clients diagnosed with alcohol dependence who are abstinent at treatment initiation.

A nursing instructor is discussing various challenges in the treatment of clients diagnosed with bipolar disorder. Which student statement demonstrates an understanding of the most critical challenge in the care of these clients?

"Treatment is compromised when clients choose not to take their medications." Rationale: The nursing student should understand that the most critical challenge in the care of clients diagnosed with bipolar disorder is that treatment is often compromised when clients choose to not take their medications. Symptoms of bipolar disorder will reemerge if medication is stopped.

A client began taking lithium for the treatment of bipolar disorder approximately 1 month ago. The client asks if it is normal to have gained 12 pounds in this time frame. Which is the appropriate nursing reply?

"Weight gain is a common but troubling side effect." Rationale: The nurse should explain to the client that weight gain is a common side effect of lithium carbonate. The nurse should educate the client on the importance of medication compliance and discuss concerns with the prescribing physician if the client does not wish to continue taking the medication.

During a counseling session with a patient diagnosed with depression, the patient states, "I know my husband doesn't love me anymore." Which response by the healthcare provider demonstrates therapeutic communication?

"What happened to make you think your husband doesn't love you anymore?"

During a counseling session with a patient diagnosed with depression, the patient states, "I know my husband doesn't love me anymore." Which response by the healthcare provider demonstrates therapeutic communication?

"What happened to make you think your husband doesn't love you anymore?" Rationale: The goal of therapeutic communication is to preserve the self-respect of the patient and caregiver. Initially, the healthcare provider will want to communicate understanding of the situation. Therapeutic communication in this situation would consist of asking a question to explore the patient's perceptions and valuing the patient's feelings.

The healthcare provider is teaching a patient diagnosed with schizophrenia about the medication clozapine (Clozaril). Which of the following will be included in the teaching? Select all that apply. Select all that apply.

"You should eat a healthy diet with plenty of fruits, vegetables, and fiber." "Let us know if you experience symptoms of infection such as fever or fatigue." "Remember to make position changes slowly until you get used to the medication." "You'll need to come in periodically so your lipid profile can be monitored." (NOT "Call our office if you experience increased thirst and increased urination." or "Remember that it's important that you avoid all citrus and citrus juices.") Rationale: Clozapine blocks a variety of receptors and can cause a range of adverse effects, including metabolic problems. Clozapine blocks muscarinic cholinergic receptors, potentially causing constipation and other anticholinergic effects. Clozapine blocks alpha-1 receptors, which increases the potential for orthostatic hypotension. Metabolic effects include weight gain, dyslipidemia, and the development of diabetes. Clozapine is associated with agranulocytosis, which may be fatal. WBC count and absolute neutrophil count (ANC) will be monitored, and patients should be advised to report symptoms of infection. Grapefruit and grapefruit juice can inhibit the cytochrome P450 isoenzyme CYP3A4, which can increase serum levels of clozapine. The patient should not be told to avoid other citrus fruits and juices.

A 16-year-old client diagnosed with schizophrenia experiences command hallucinations to harm others. The client's parents ask a nurse, "Where do the voices come from?" Which is the appropriate nursing reply?

"Your child has a chemical imbalance of the brain, which leads to altered thoughts."

A 16-year-old client diagnosed with schizophrenia experiences command hallucinations to harm others. The client's parents ask a nurse, "Where do the voices come from?" Which is the appropriate nursing reply?

"Your child has a chemical imbalance of the brain, which leads to altered thoughts." Rationale: The nurse should explain that a chemical imbalance of the brain leads to altered thought processes. Hallucinations, or false sensory perceptions, may occur in all five senses. The client who hears voices is experiencing an auditory hallucination.

A client's wife has been making excuses for her alcoholic husband's work absences. In family therapy, she states, "I just need to work harder to get him there on time." Which is the appropriate nursing response?

"Your husband needs to deal with the consequences of his drinking." Rationale: The appropriate nursing response is to use confrontation with caring. In Stage One (The Survival Stage) of recovery from codependency, the codependent person must begin to let go of the denial that problems exist or that his or her personal capabilities are unlimited.

A client refuses to go on a cruise to the Bahamas with his spouse because of fearing that the cruise ship will sink and all will drown. Using a cognitive theory perspective, the nurse should use which of these statements to explain to the spouse the etiology of this fear?

"Your spouse may be experiencing a distorted and unrealistic appraisal of the situation." Rationale: The nurse should explain that from a cognitive perspective the client is experiencing a distorted and unrealistic appraisal of the situation. From a cognitive perspective, fear is described as the result of faulty cognitions.

Which statement made by the student nurse regarding nutrition and symptoms of depression would require correction

"my client will need to avoid protein if he or she wants to avoid symptoms of depression "

A client with mania has shown progressive improvement with lithium therapy. Which suggestions would the nurse give to the client? Select all that apply

-Notify the healthcare provider if pregnancy is planned or suspected -Contact HCP if you have excessive vomitting

The nurse is caring for a client who is diagnosed with hypomania. Which behavior does the nurse find in the client? Select all that apply

-The client is cheerful and expansive with an underlying irritability -The client talks and laughs very loudly while communicating with the nurse

Which interventions would the nurse implement while applying a selegiline transdermal patch? Select all that apply.

-Wash hands after applying -avoid exposure to the site of application to direct heat -apply a new patch to a new site if the patch falls off

14. A child diagnosed with severe autistic spectrum disorder has the nursing diagnosis disturbed personal identity. Which outcome would best address this client diagnosis? 1. The client will name own body parts as separate from others by day five. 2. The client will establish a means of communicating personal needs by discharge. 3. The client will initiate social interactions with caregivers by day four. 4. The client will not harm self or others by discharge.

1

15. A nursing instructor presents a case study in which a three-year-old child is in constant motion and is unable to sit still during story time. She asks a student to evaluate this child's behavior. Which student response indicates an appropriate evaluation of the situation? 1. "This child's behavior must be evaluated according to developmental norms." 2. "This child has symptoms of attention deficit-hyperactivity disorder." 3. "This child has symptoms of the early stages of autistic disorder." 4. "This child's behavior indicates possible symptoms of oppositional defiant disorder."

1

5. After an adolescent diagnosed with attention deficit-hyperactivity disorder (ADHD) begins methylphenidate (Ritalin) therapy, a nurse notes that the adolescent loses 10 pounds in a 2-month period. What is the best explanation for this weight loss? 1. The pharmacological action of Ritalin causes a decrease in appetite. 2. Hyperactivity seen in ADHD causes increased caloric expenditure. 3. Side effects of Ritalin cause nausea, and, therefore, caloric intake is decreased. 4. Increased ability to concentrate allows the client to focus on activities rather than food.

1

6. A nurse assesses an adolescent client diagnosed with conduct disorder who, at the age of 8, was sentenced to juvenile detention. How should the nurse interpret this assessment data? 1. Childhood-onset conduct disorder is more severe than the adolescent-onset type, and these individuals likely develop antisocial personality disorder in adulthood. 2. Childhood-onset conduct disorder is caused by a difficult temperament, and the child is likely to outgrow these behaviors by adulthood. 3. Childhood-onset conduct disorder is diagnosed only when behaviors emerge before the age of 5, and, therefore, improvement is likely. 4. Childhood-onset conduct disorder has no treatment or cure, and children diagnosed with this disorder are likely to develop progressive oppositional defiant disorder.

1

Which is the normal therapeutic serum concentration in a client with acute mania receiving lithium therapy

1.0 mEq

A client has been taking lithium for several years with good symptom control. The client presents in the emergency department with blurred vision, tinnitus, and severe diarrhea. The nurse should correlate these symptoms with which lithium level?

1.7 mEq/L Rationale: The therapeutic level of lithium carbonate is 1.0 to 1.5 mEq/L for acute mania and 0.6 to 1.2 mEq/L for maintenance therapy. There is a narrow margin between the therapeutic and toxic levels. The symptoms presented in the question can be correlated with a lithium level of 1.7 mEq/L. Levels of 2.3 mEq/L and 3.7 mEq/L would produce more extreme symptoms of intensified toxicity, eventually leading to death.

A nurse caring for a manic client receiving lithium for maintenance therapy observes that the client has persistent gastrointestinal upset and blurred vision. After reviewing the laboratory reports, the nurse finds that the client has signs of lithium toxicity. Which concentration of lithium does the nurse find in the client's blood serum?

1.8 mEq/L (>1.2)

A nurse is reviewing STAT laboratory data of a client presenting in the emergency department. At what minimum blood alcohol level should a nurse expect intoxication to occur?

100 mg/dL Rationale: The nurse should expect that 100 mg/dL is the minimum blood alcohol level at which intoxication occurs. Intoxication usually occurs between 100 and 200 mg/dL. Death has been reported at levels ranging from 400 to 700 mg/dL.

Which range is the daily dose range of lithium carbonate for a client with acute mania

1800-2400mg

10. A preschool child is admitted to a psychiatric unit with the diagnosis autistic spectrum disorder. To help the child feel more secure on the unit, which intervention should a nurse include in this client's plan of care? 1. Encourage and reward peer contact. 2. Provide consistent caregivers. 3. Provide a variety of safe daily activities. 4. Maintain close physical contact throughout the day.

2

13. Which behavioral approach should a nurse use when caring for children diagnosed with disruptive behavior disorders? 1. Involving parents in designing and implementing the treatment process 2. Reinforcing positive actions to encourage repetition of desirable behaviors 3. Providing opportunities to learn appropriate peer interactions 4. Administering psychotropic medications to improve quality of life

2

17. A physician orders methylphenidate (Ritalin) for a child diagnosed with ADHD. Which information about this medication should the nurse provide to the parents? 1. If one dose of Ritalin is missed, double the next dose. 2. Administer Ritalin to the child after breakfast. 3. Administer Ritalin to the child just prior to bedtime. 4. A side effect of Ritalin is decreased ability to learn.

2

3. A child has been diagnosed with autistic spectrum disorder. The distraught mother cries out, "I'm such a terrible mother. What did I do to cause this?" Which nursing response is most appropriate? 1. "Researchers really don't know what causes autistic spectrum disorder, but the relationship between autistic disorder and fetal alcohol syndrome is being explored." 2. "Poor parenting doesn't cause autistic spectrum disorder. Research has shown that abnormalities in brain structure or function are to blame. This is beyond your control." 3. "Research has shown that the mother appears to play a greater role in the development of autistic spectrum disorder than the father." 4. "Lack of early infant bonding with the mother has shown to be a cause of autistic spectrum disorder. Did you breastfeed or bottle-feed?"

2

7. Which finding should a nurse expect when assessing a child diagnosed with separation anxiety disorder? 1. The child has a history of antisocial behaviors. 2. The child's mother is diagnosed with an anxiety disorder. 3. The child previously had an extroverted temperament. 4. The child's mother and father have an inconsistent parenting style.

2

22. Which of the following findings should a nurse identify that would contribute to a client's development of ADHD? (Select all that apply.) 1. The client's father was a smoker. 2. The client was born 7 weeks premature. 3. The client is lactose intolerant. 4. The client has a sibling diagnosed with ADHD. 5. The client has been diagnosed with dyslexia.

2, 4

11. A preschool child diagnosed with autistic spectrum disorder has been engaging in constant head-banging behavior. Which nursing intervention is appropriate? 1. Place client in restraints until the aggression subsides. 2. Sedate the client with neuroleptic medications. 3. Hold client's head steady and apply a helmet. 4. Distract the client with a variety of games and puzzles.

3

18. Which should be the priority nursing intervention when caring for a child diagnosed with conduct disorder? 1. Modify environment to decrease stimulation and provide opportunities for quiet reflection. 2. Convey unconditional acceptance and positive regard. 3. Recognize escalating aggressive behavior and intervene before violence occurs. 4. Provide immediate positive feedback for appropriate behaviors

3

19. A mother questions the decreased effectiveness of methylphenidate (Ritalin), prescribed for her child's ADHD. Which nursing response best addresses the mother's concern? 1. "The physician will probably switch from Ritalin to a central nervous system stimulant." 2. "The physician may prescribe an antihistamine with the Ritalin to improve effectiveness." 3. "Your child has probably developed a tolerance to Ritalin and may need a higher dosage." 4. "Your child has developed sensitivity to Ritalin and may be exhibiting an allergy."

3

4. In planning care for a child diagnosed with autistic spectrum disorder, which would be a realistic client outcome? 1. The client will communicate all needs verbally by discharge. 2. The client will participate with peers in a team sport by day four. 3. The client will establish trust with at least one caregiver by day five. 4. The client will perform most self-care tasks independently.

3

20. After studying the DSM-5 criteria for oppositional defiant disorder (ODD), which listed symptom would a student nurse recognize? 1. Arguing and annoying older sibling over the past year 2. Angry and resentful behavior over a three-month period 3. Initiating physical fights for more than 18 months 4. Arguing with authority figures for more than six months

4

Which condition would be the most common comorbid condition with bipolar disorder among children and adolescents

ADHD

At which age would a child experience symptoms of depression that include morbid thoughts, excessive worrying, and poor self-esteem

9-12 years old

Which should the nurse recognize as an example of localized amnesia?

A client cannot remember events surrounding a fatal car accident.

Which client is a nurse most likely to admit to an inpatient facility for self-destructive behaviors?

A client diagnosed with borderline personality disorder

Which client's condition can be best explained using the psychoanalytical theory of depression?

A client who is experiencing depression after a sudden death of a loved one

The healthcare provider is teaching a group of students about suicide assessment and prevention. Which of the following will be included in the teaching? Select all that apply.

A patient who talks about suicide may be signaling others for help. When medication improves a patient's mood, they may attempt suicide. Rationale: When assessing suicide risk the healthcare provider will ask if the patient is having suicidal thoughts. Talking about suicide does not increase suicide risk. Often a patient contemplating suicide will give verbal or nonverbal signals about their intention to harm themselves, but in rare cases a suicide can occur without warning. The strongest single factor predictive of suicide is a history of attempted suicide. The risk of suicide may actually increase, especially during the initial phase of treatment, and more often in children, adolescents, and young adults. Patients should be observed closely for changes in behavior or mood that may indicate suicidality.

A client diagnosed with chronic alcohol use disorder is being discharged from an inpatient treatment facility after detoxification. Which client outcome related to Alcoholics Anonymous (AA) would be most appropriate for a nurse to discuss with the client during discharge teaching?

After discharge, the client will immediately attend 90 AA meetings in 90 days. Rationale: The most appropriate client outcome for the nurse to discuss during discharge teaching is attending 90 AA meetings in 90 days after discharge. AA is a major self-help organization for the treatment of alcoholism. It accepts alcoholism as an illness and promotes total abstinence as the only cure.

A client with multiple sclerosis has depression. Which symptoms can indicate underlying depression in this client

Agitation and restlessness

A college student is unable to take a final examination because of severe test anxiety. Instead of studying, the student relieves stress by attending a movie. Which priority nursing diagnosis should a campus nurse assign for this client?

Altered coping r/t anxiety Rationale: The priority nursing diagnosis for this client is altered coping R/T anxiety. The nurse should assist in implementing interventions that should improve the client's healthy coping skills and reduce anxiety.

A client diagnosed with bipolar I disorder is distraught over insomnia experienced over the last 3 nights and a 12-pound weight loss over the past 2 weeks. Which should be this client's priority nursing diagnosis?

Altered nutrition: less than body requirements R/T hyperactivity AEB weight loss Rationale: The nurse should identify that the priority nursing diagnosis for this client is altered nutrition: less than body requirements R/T hyperactivity AEB weight loss. Due to the client's rapid weight loss, the nurse should prioritize interventions to ensure proper nutrition and health.

A patient diagnosed with agoraphobia is scheduled for a functional magnetic resonance imaging (fMRI) study of the brain. The healthcare provider anticipates that the scan will show increased activity in which of the following areas of this patient's brain?

Amygdala Rationale: Nerve fibers link this portion of the brain to the limbic system and frontal cortex. This part of the brain is a component of the patient's "fear circuitry." Patients diagnosed with anxiety disorders often demonstrate hyperactivity of the amygdala, insula, and limbic system.

What is the main difference between an individual diagnosed with bipolar I and bipolar II?

An individual diagnosed with bipolar II has never had a manic episode. An individual diagnosed with bipolar I disorder has had at least one manic episode.

While assessing a client for symptoms of depression, the psychiatrist assigns a score of three for psychic anxiety according to the Hamilton Depression Rating Scale. Which observation in the client enables the psychiatrist to give the score

Apprehensive attitude apparent in the face and speech

What is the main difference between an individual diagnosed with bipolar I and bipolar II?

An individual diagnosed with bipolar II has never had a manic episode. An individual diagnosed with bipolar I disorder has had at least one manic episode. Rationale: A hypomanic episode is a period of abnormally elevated mood and abnormally increased energy lasting at least 4 consecutive days; however, the episode is not severe enough to cause impairment in functioning or to require hospitalization. A manic episode is a period of abnormally elevated mood and abnormally increased energy lasting at least 7 consecutive days; however, the episode is severe enough to cause impairment in functioning or to require hospitalization. Although it is possible for an individual diagnosed with bipolar I to never experience hypomania, it is extremely unlikely. An individual diagnosed with bipolar II has never had a manic episode. An individual diagnosed with bipolar I disorder has had at least one manic episode.

When assessing a patient diagnosed with schizophrenia, which of the following will the healthcare provider identify as a negative symptom?

Anhedonia

When assessing a patient diagnosed with schizophrenia, which of the following will the healthcare provider identify as a negative symptom?

Anhedonia Rationale: Schizophrenia is characterized by symptoms which are classified as either positive or negative. Positive symptoms of schizophrenia are an exaggeration or distortion of normal functioning. Symptoms are classified as negative when they demonstrate a decrease of normal functioning such as flat affect (the inability to show emotion), anhedonia (the inability to experience pleasure), or asociality (an inability to maintain social contacts). Positive symptoms include hallucinations, delusions, or disorganized speech.

Over the past year, a woman has cooked gourmet meals for her family but eats only tiny servings. She wears layered, loose clothing and now has amenorrhea. Her current weight is 95 pounds, a loss of 35 pounds. Which medical diagnosis is most likely?

Anorexia nervosa

While talking to a female client diagnosed with schizophrenia, the nurse notices the client looks away from the nurse and stares at the wall while making facial grimaces. What is the most appropriate action by the nurse?

Ask the client if she is seeing something in the wall. Rationale: This client is experiencing visual hallucinations. This nursing response is an example of "voicing or stating the obvious," which facilitates the patient to verbalize and validate the observation. This should be followed by presentation of reality and knowing the content of the hallucination.

Which reaction to a compliment from another client should a nurse identify as a typical response from a client diagnosed with avoidant personality disorder?

Being grateful for the compliment but fearing later rejection and humiliation

A client with a history of heavy alcohol use is brought to an emergency department (ED) by family members who state that the client has had nothing to drink in the last 24 hours. Which client symptom should the nurse immediately report to the ED physician?

Blood pressure of 180/100 mm Hg Rationale: The nurse should recognize that high blood pressure is a symptom of alcohol withdrawal and should promptly report this finding to the physician. Complications associated with alcohol withdrawal may progress to alcohol withdrawal delirium and possible seizure activity on about the second or third day following cessation of prolonged alcohol consumption.

Which side effect is suspected in the client who is on verapamil therapy

Bradycardia

Which medication indicated for attention deficit-hyperactivity disorder (ADHD) induces mania

Bupropion

A patient diagnosed with anorexia nervosa has a body mass index (BMI) of 14.8 kg/m2. Which assessment finding is most likely to accompany this value?

Cachexia

A client presents in the emergency department with complaints of overwhelming anxiety. Which of the following is a priority for the nurse to assess?

Cardiac status Rationale: Although all of the listed aspects of assessment are important, the priority is to evaluate cardiac status since a person having an MI, CHF, or mitral valve prolapse can present with symptoms of anxiety.

The nursing care plan for a patient diagnosed with anorexia nervosa includes the intervention Monitor for complications of re-feeding. Which body system should a nurse closely monitor for dysfunction?

Cardiovascular

A client with bipolar disorder is diagnosed with glaucoma. Which medication is contraindicated in this client for the treatment of bipolar disorder

Clonazepam

Which action would the nurse take before administering lithium carbonate to a client for the first time

Check for history of seizures

Which of the following explanations should a nurse include when teaching parents why is it difficult to diagnose a child or adolescent exhibiting symptoms of bipolar disorder? Select all that apply.

Children are naturally active, energetic, and spontaneous. Bipolar symptoms are similar to attention deficit-hyperactivity disorder symptoms. Rationale: It is difficult to diagnose a child or adolescent with bipolar disorder because bipolar symptoms are similar to attention deficit-hyperactivity disorder symptoms and because children are naturally active, energetic, and spontaneous. Symptoms may also be comorbid with other childhood disorders, such as conduct disorder.

Which medication orders should a nurse anticipate for a client who has a history of complicated withdrawal from benzodiazepines?

Chlordiazepoxide (Librium) and phenytoin (Dilantin) Rationale: The nurse should anticipate that a physician would order chlordiazepoxide (Librium) and phenytoin (Dilantin) for a client who has a history of complicated withdrawal from benzodiazepines. It is common for long-lasting benzodiazepines to be prescribed for substitution therapy. Phenytoin (Dilantin) is an anticonvulsant that would be indicated for a client who has experienced a complicated withdrawal. Complicated withdrawals may progress to seizure activity.

After reviewing the medication list of a client with bipolar mania, the nurse advises the client to use extra sunscreen, wear protective clothing, and limit time spent outdoors to 30 minutes. Which medication did the nurse likely find among the client's prescriptions?

Chlorpromazine

When talking with a patient diagnosed with schizophrenia, the healthcare provider notes the patient continually states, "I'm the man with a plan, yes I am." The healthcare provider will document this behavior as which of the following?

Clang associations Rationale: All of these are manifestations of disorganized thought often seen in a patient diagnosed with schizophrenia. Word salad occurs when real words are linked together without any logical connection. This patient is demonstrating clanging, or clang associations, which is characterized by linking together words that rhyme or sound alike.

The nurse is caring for a client who lost a brother in a plane crash. Which statement indicates effectiveness of the nursing care

Client expresses anger about the loss

A client with depression tells the nurse, "I am a loser and have not achieved anything big in life. My family and friends think that I am worthless." Which therapy does the nurse anticipate will be prescribed by the primary health-care provider to help in improving the mood of the client?

Cognitive therapy

Which therapy is beneficial to a client who frequently expresses the thought that he or she is a complete failure and feels depressed

Cognitive therapy

The registered nurse is teaching a group of student nurses about cognitive therapy. Which statement made by a student nurse requires correction

Cognitive therapy describes mania as a triad of negative distortions

A client diagnosed with schizophrenia states, "My psychiatrist is out to get me. I'm sad that the voice is telling me to stop him." What symptom is the client exhibiting, and what is the nurse's legal responsibility related to this symptom?

Command hallucinations; warn the psychiatrist Rationale: The nurse should determine that the client is exhibiting command hallucinations. The nurse's legal responsibility is to warn the psychiatrist of the potential for harm. A client who is demonstrating a risk for violence could potentially become physically, emotionally, and/or sexually harmful to others or to self.

Which information would the nurse provide to the client to prevent persistent vomiting and diarrhea for a client with bipolar disorder who is on lithium carbonate therapy

Consume a diet with adequate sodium

Neurological tests have ruled out pathology in a client's sudden lower-extremity paralysis. Which nursing care should be included for this client?

Deal with physical symptoms in a detached manner.

A patient diagnosed with depression is prescribed fluoxetine (Prozac). Which of the following would the healthcare provider most likely observe if the patient experiences an adverse effect of this medication?

Decreased libido

A patient diagnosed with depression is prescribed fluoxetine (Prozac). Which of the following would the healthcare provider most likely observe if the patient experiences an adverse effect of this medication?

Decreased libido Rationale: Fluoxetine is a selective serotonin reuptake inhibitor (SSRI). Fluoxetine increases the synaptic concentration of serotonin the central nervous system, but may have effects on other nervous system functions. Although the mechanism has not been completely elucidated, sexual dysfunction is one of the most common adverse effects of SSRIs in both men and women.

A nurse is assessing a client diagnosed with schizophrenia. The nurse asks the client, "Do you receive special messages from certain sources, such as the television or radio?" Which potential symptom of this disorder is the nurse assessing?

Delusions of reference

A nurse is assessing a client diagnosed with schizophrenia. The nurse asks the client, "Do you receive special messages from certain sources, such as the television or radio?" Which potential symptom of this disorder is the nurse assessing?

Delusions of reference Rationale: The nurse is assessing for the potential symptom of delusions of reference. A client who believes that he or she receives messages through the radio is experiencing delusions of reference. When a client experiences these delusions, he or she interprets all events within the environment as personal references.

How would a nurse differentiate a client diagnosed with panic disorder from a client diagnosed with generalized anxiety disorder (GAD)?

Depersonalization is commonly seen in panic disorder and absent in GAD. Rationale: The nurse should recognize that a client diagnosed with panic disorder experiences depersonalization, whereas a client diagnosed with GAD would not. Depersonalization refers to being detached from oneself when experiencing extreme anxiety.

A depressed client reports to a nurse a history of divorce, job loss, family estrangement, and cocaine abuse. According to learning theory, what is the cause of this client's symptoms?

Depression is a result of repeated failures. Rationale: Learning theory describes a model of "learned helplessness" in which multiple life failures cause the client to abandon future attempts to succeed.

An adolescent in a psychiatric unit has a temperamental outburst, saying, "I tried my best but could not reach my parents' expectations. I disappointed them. My life is of no use." Which condition would the nurse suspect in the client?

Depressive mood dysregulation disorder

Which antidepressant drug acts as a serotonin-norepinephrine reuptake inhibitor

Desvenlafaxine

The nurse is caring for a client with depression. Which primary nursing intervention is appropriate for this client

Develop a trusting relationship with the client

The primary health-care provider prescribes olanzapine to a client who is experiencing acute mania. Which condition in the health history of the client indicates the risk for complications

Diabetes mellitus

Emergency medical personnel bring a patient to the emergency department. The patient reports overdosing on sertraline (Paxil) in a suicide attempt. Which of these would the healthcare provider identify as consistent with serotonin syndrome? Select all that apply.

Diaphoresis Gastrointestinal distress Muscle rigidity Tachycardia

A client is diagnosed with illness anxiety disorder. Which of the following symptoms is the client most likely to exhibit? Select all that apply.

Disabling fear of having a serious illness Obsessive-compulsive traits Depression

Which signs and symptoms indicate that the client has disturbed sensory perception? Select all that apply

Disorentation Auditory hallucinations

A nurse is working with a client diagnosed with somatic symptom disorder. What predominant symptoms should a nurse expect to assess?

Disproportionate and persistent thoughts about the seriousness of one's symptoms

Which symptom displayed by the client may indicate hypomanic episodes of bipolar disorder

Distractibility

Which neurotransmitter level in the mesolimbic system of the brain is thought to exert a strong influence over human mood and behavior

Dopamine

The primary health-care provider prescribes lithium carbonate therapy to a client with mania. Which instruction would the nurse provide to the client to prevent adverse effects associated with the medication

Drink 6-8 large glassess of water each day

A nurse, on reviewing the prescription of a client with depression, instructs the client to chew sugarless candies occasionally. Which complication can be relieved in the client by implementing the nurse's suggestion

Dry mouth

How does electroconvulsive therapy (ECT) work to treat a depressed clien

ECT increases serotonin levels

Why is the dose of antidepressant drugs decreased in elderly clients

Elderly people have decreased elimination of drugs

Which action would the nurse take during the first phase of individual psychotherapy?

Encourage the client to continue participating in regular activities

A nurse has been caring for a client diagnosed with generalized anxiety disorder (GAD). Which of the following nursing interventions would address this client's symptoms? (Select all that apply.)

Encourage the client to recognize the signs of escalating anxiety. Encourage the client to cognitively reframe thoughts about situations that generate anxiety. Encourage the client to employ newly learned relaxation techniques. Encourage the client to avoid caffeinated products. Rationale: Nursing interventions that address GAD symptoms should include encouraging the client to recognize signs of escalating anxiety, to employ relaxation techniques, to cognitively reframe thoughts about anxiety-provoking situations, and to avoid caffeinated products. Avoiding situations that cause stress is not an appropriate intervention, because avoidance does not help the client overcome anxiety. Stress is a component of life and is not easily evaded.

Which statement made by the client would be an example of an "all-or-nothing" statement

Everything I do is great

A patient is admitted to an inpatient psychiatric unit because of a plan to commit suicide by taking an overdose of medication. When administering medications to this patient, which of these interventions is the priority?

Ensure that the patient is not "cheeking" the medications

A patient is admitted to an inpatient psychiatric unit because of a plan to commit suicide by taking an overdose of medication. When administering medications to this patient, which of these interventions is the priority?

Ensure that the patient is not "cheeking" the medications Rationale: All of these interventions may be included in the plan of care, but one of these is the priority. The priority intervention is designed to increase patient safety. A patient who has suicidal ideation, especially by overdosing on medications, should be monitored for "cheeking." Cheeking occurs when a patient hides the medication in the mouth, and hoards it so it can be used for the suicide attempt.

Which conditions can be effectively treated using topiramate? Select all that apply

Epilepsy Mirgraines

An inpatient client is newly diagnosed with dissociative identity disorder (DID) stemming from severe childhood sexual abuse. Which nursing intervention takes priority?

Establish trust and rapport

What tool should a nurse use to differentiate occasional spontaneous behaviors of children from behaviors associated with bipolar disorder?

FIND" tool

Patients with anxiety disorders often show decreased amygdala response to anxiety cues.

False Rationale: Amygdala response is increased or heightened in anxiety.

Benzodiazepines are the drugs of choice in the treatment of anxiety disorders.

False Rationale: Antidepressants are first line, benzos should be used for acute symptoms

Buspar has high risk for abuse and dependence.

False Rationale: Buspirone differs from typical benzodiazepine anxiolytics in that it does not exert anticonvulsant or muscle relaxant effects. It also lacks the prominent sedative effect that is associated with more typical anxiolytics.

Obsessive-compulsive disorder is considered an anxiety disorder.

False Rationale: Obsessive-compulsive disorder (included in the obsessive-compulsive and related disorders), acute stress disorder, and posttraumatic stress disorder (included in the trauma and stress-related disorders) are no longer considered anxiety disorders under DSM 5.

The main indication for Buspar is Panic disorders and Phobias.

False Rationale: The efficacy of Buspar has been demonstrated in controlled clinical trials of outpatients whose diagnosis roughly corresponds to Generalized Anxiety Disorder (GAD).

The cognitive theory has explained anxiety as a conflict between the id and ego.

False Rationale: This explanation comes from psychoanalytic/psychodynamic theory

The diagnosis of general anxiety disorder is made primarily by history.

False Rationale: This is panic disorder.

Patients with general anxiety disorder frequently present to the emergency department (ED) with chest pain or dyspnea, fearing that they are dying of myocardial infarction.

False Rationale: This is patients with panic disorder

A nurse is planning care for a child who is experiencing depression. Which medication is approved by the U.S. Food and Drug Administration (FDA) for the treatment of depression in children and adolescents?

Fluoxetine (Prozac)

A client is diagnosed with persistent depressive (dysthymia) disorder. Which should a nurse classify as an affective symptom of this disorder?

Gloomy and pessimistic outlook on life Rationale: The nurse should classify a gloomy and pessimistic outlook on life as an affective symptom of dysthymia. Symptoms of depression can be described as alterations in four areas of human functions: affective, behavioral, cognitive, and physiological. Affective symptoms are those that relate to the mood.

A nurse is planning care for a child who is experiencing depression. Which medication is approved by the U.S. Food and Drug Administration (FDA) for the treatment of depression in children and adolescents?

Fluoxetine (Prozac) Rationale: Fluoxetine (Prozac) is FDA approved for the treatment of depression in children and adolescents. Fluoxetine is a selective serotonin reuptake inhibitor (SSRI) used in the treatment of depression. All antidepressants carry an FDA warning for increased risk of suicide in children and adolescents.

The healthcare provider is teaching a group of students about the biological basis of schizophrenia. Which of the following will be included in the teaching? Select all that apply.

GABAergic interneuron dysregulation Family history of schizophrenia Increased dopamine levels Decreased norepinephrine levels Prenatal exposure to influenza (NOT stimulation of the amygdala) Rationale: Although the exact cause of schizophrenia is unknown, gene-environment interactions are implicated. Having a family member diagnosed with schizophrenia is a risk factor for developing schizophrenia. Perinatal complications such and maternal exposure to influenza during pregnancy are associated with an increased incidence of schizophrenia. An imbalance between excitation (via glutamate, the major CNS excitatory neurotransmitter), and GABA (the major CNS inhibitory neurotransmitter) in the cerebral cortex may be responsible for some of the pathophysiological processes involved in schizophrenia. Because antipsychotic medications block the dopaminergic D2 receptor in the central nervous system, excess dopamine has been identified as a potential cause of the psychotic symptoms associated with schizophrenia.

Emergency medical personnel bring a patient to the emergency department. The patient reports overdosing on sertraline (Paxil) in a suicide attempt. Which of these would the healthcare provider identify as consistent with serotonin syndrome? Select all that apply.

Gastrointestinal distress Diaphoresis Muscle rigidity Tachycardia (NOT facial grimacing or hypothermia) Rationale: Serotonin is a neurotransmitter that has numerous roles throughout the body. Serotonin syndrome is caused by overstimulation of serotonin receptors in the central and peripheral nervous systems. Clinical features of serotonin syndrome include agitation, diaphoresis, tachycardia, hypertension, hyperthermia, gastrointestinal distress, and hyperreflexia.

Which food item is safe to be consumed by a client who is on monoamine oxidase inhibitor (MAOI) therapy

Grilled chicken breast

A client is admitted for alcohol detoxification. During detoxification, which symptoms should the nurse expect to assess?

Gross tremors, delirium, hyperactivity, and hypertension

A client is admitted for alcohol detoxification. During detoxification, which symptoms should the nurse expect to assess?

Gross tremors, delirium, hyperactivity, and hypertension Rationale: Withdrawal is defined as the physiological and mental readjustment that accompanies the discontinuation of an addictive substance. Symptoms can include gross tremors, delirium, hyperactivity, hypertension, nausea, vomiting, tachycardia, hallucinations, and seizures.

Laboratory results reveal elevated levels of prolactin in a client diagnosed with schizophrenia. When assessing the client, the nurse should expect to observe which symptoms? Select all that apply.

Gynecomastia Galactorrhea (NOT apathy, anhedonia, or social withdrawal) Rationale: Dopamine blockage, an expected action of antipsychotic medications, also results in prolactin elevation. Galactorrhea and gynecomastia are symptoms of prolactin elevation.

A client diagnosed with major depressive disorder states, "I've been feeling 'down' for 3 months. Will I ever feel like myself again?" Which reply by the nurse will best assess this client's affective symptoms?

Help me understand what you mean when you say, 'feeling down'?"

Looking at a slightly bleeding paper cut, the client screams, "Somebody help me, quick! I'm bleeding. Call 911!" A nurse should identify this behavior as characteristic of which personality disorder?

Histrionic personality disorder

A client diagnosed with schizoaffective disorder is admitted for social skills training. Which information should be taught by the nurse?

How to make eye contact when communicating

A client diagnosed with schizoaffective disorder is admitted for social skills training. Which information should be taught by the nurse?

How to make eye contact when communicating Rationale: The nurse should plan to teach the client how to make eye contact when communicating. Social skills, such as making eye contact, can assist clients in communicating needs and maintaining connectedness.

Which behavior of a client made the nurse implement keeping juices and snacks on the unit at all times

Hyperactivity

A patient is diagnosed with mild depression. Which of the following describe a physiological alteration often associated with this diagnosis? Select all that apply.

Hypersomnia Insomnia Anorexia (NOT difficulty concentrating or amenorrhea) Rationale: Physiological symptoms associated with depression tend to become more intense as depression worsens. Physiological symptoms associated with mild depression are often the same as with normal grieving. Anorexia, hypersomnia, insomnia are examples of physical alterations associated with mild depression. Amenorrhea is a physiological alteration associated with severe depression. Difficulty concentrating is a cognitive alteration.

Which side effects are commonly seen in clients taking monoamine oxidase inhibitors (MAOIs)? Select all that apply.

Hypertensive crisis Application site reaction

A patient diagnosed with anorexia nervosa virtually stopped eating 5 months ago and has lost 25% of body weight. A nurse asks, describe what you think about your present weight and how you look. Which response by the patient is most consistent with the diagnosis?

I am fat and ugly.

While communicating with a client, the nurse suspects that the client is having a manic episode of bipolar disorder. Which statement of the client supports the nurse's suspicion

I communicate better than anyone

The nurse is caring for a client who is a musician who has gone into a state of depression after the death of his or her spouse. The nurse tells the client, "You need to refocus on your music." Which statement made by the client after a few days indicates effective nursing intervention

I will now be able to record good music for my upcoming album

Which statement made by the student nurse requires correction regarding promoting client safety

I will use the most restrictive method of promitng safety

Which statement is a nurse most likely to hear from a patient diagnosed with anorexia nervosa?

I would be happy if I could lose 20 more pounds.

Upon admission for symptoms of alcohol withdrawal, a client states, "I haven't eaten in 3 days." Assessment reveals BP 170/100 mm Hg, P 110, R 28, and T 97∘∘F (36∘∘C) with dry skin, dry mucous membranes, and poor skin turgor. What should be the priority nursing diagnosis?

Imbalanced nutrition: less than body requirements

Upon admission for symptoms of alcohol withdrawal, a client states, "I haven't eaten in 3 days." Assessment reveals BP 170/100 mm Hg, P 110, R 28, and T 97LaTeX: ^\circ ∘ F (36LaTeX: ^\circ ∘ C) with dry skin, dry mucous membranes, and poor skin turgor. What should be the priority nursing diagnosis?

Imbalanced nutrition: less than body requirements Rationale: The nurse should assess that the priority nursing diagnosis is imbalanced nutrition: less than body requirements. The client is exhibiting signs and symptoms of malnutrition as well as alcohol withdrawal. The nurse should consult a dietitian, restrict sodium intake to minimize fluid retention, and provide small, frequent feedings of nonirritating foods.

A client reports dry mouth, blurred vision, constipation, and urinary retention after taking the medication prescribed by the primary health-care provider to treat depression. Which medication does the nurse find in the client's prescription

Imipramine (tricyclic antidepressant)

A depressed patient with a history of three suicide attempts has been taking fluoxetine (Prozac) for 1 week. The client suddenly presents with a bright affect, rates mood at 9/10, and is much more communicative. Which action should be the nurse's priority at this time?

Increase frequency of client observation Rationale: Suicide risk may occur early during treatment with antidepressants. The nurse should be aware that a sudden increase in mood rating and change in affect could indicate that the client is at risk for suicide and client observation should be more frequent. The return of energy may bring about an increased ability to act out self-destructive behaviors prior to attaining the full therapeutic effect of the antidepressant medication.

The healthcare provider is teaching a group of students about the biological basis of schizophrenia. Which of the following will be included in the teaching? Select all that apply.

Increased dopamine levels Decreased norepinephrine levels Family history of schizophrenia Prenatal exposure to influenza GABAergic interneuron dysregulation

A patient diagnosed with bipolar disorder is experiencing the manic phase of the disorder. Which neurotransmitter alterations will the healthcare provider identify as contributing to mania? Select all that apply.

Increased glutamate Decreased GABA Increased norepinephrine Rationale: Manic episode is associated with increased glutamate (excitatory neurotransmitter), increased dopamine and norepinephrine, and decreased GABA (inhibitory neurotransmitter).

Which electrolyte imbalance may be present in a client with depression

Increased level of potassium

Which behavioral characteristics does the nurse observe in a client with hypomania

Increased perception of environment

A 19-year-old client has been admitted to the ED after snorting heroin with her boyfriend. She has pinpoint pupils and is unconscious. The client's priority nursing diagnosis is:

Ineffective breathing pattern

A 19-year-old client has been admitted to the ED after snorting heroin with her boyfriend. She has pinpoint pupils and is unconscious. The client's priority nursing diagnosis is:

Ineffective breathing pattern Rationale: Heroin intoxication is indicated by pinpoint pupils and decreased respirations. This is the priority diagnosis (option 1). Option 2, impaired fluid imbalance, and option 3, impaired physical mobility, may be important at some time but not initially. Option 4 is not a priority at this time.

As a patient admitted to the eating disorders unit undresses, a nurse observes that the patient's body is covered by fine, downy hair. The patient weighs 70 pounds and is 5 feet, 4 inches tall. Which condition should be documented?

Lanugo

A patient is diagnosed with mild depression. Which of the following describe a physiological alteration often associated with this diagnosis? Select all that apply.

Insomnia Anorexia Hypersomnia

A patient who overdosed on oxycodone is given naloxone. When assessing the patient, the healthcare provider would anticipate which of these clinical manifestations of opioid withdrawal?

Irritability and nausea Rationale: Naloxone is given to the patient in order to reverse the effects of the oxycodone. Naloxone, an opioid antagonist, will displace opioids at the opioid receptor site. The healthcare provider would expect to observe irritability and nausea. Heart rate and blood pressure will be baseline or elevated, and temperature will be unchanged. Depressed respirations and somnolence are signs of opioid intoxication.

A client recently discharged from an alcohol rehabilitation program is brought to the hospital in a state of prostration with severe throbbing headache, tachycardia, a beet-red face, dyspnea, and continuous vomiting. The client's significant other states the client got sick about 15 minutes after drinking a glass of wine. The nurse should be guided in assessment by the suspicion that the client:

Is reacting to disulfiram (Antabuse)

A client recently discharged from an alcohol rehabilitation program is brought to the hospital in a state of prostration with severe throbbing headache, tachycardia, a beet-red face, dyspnea, and continuous vomiting. The client's significant other states the client got sick about 15 minutes after drinking a glass of wine. The nurse should be guided in assessment by the suspicion that the client:

Is reacting to disulfiram (Antabuse) Rationale: The alcohol deterrent drug, Antabuse, causes a flush reaction; therefore option 3 is correct. Alcohol intoxication (option 1), stroke (option 2) and cross dependence (option 4) do not present with the listed prostration symptoms.

A client with transient depression tells the nurse, "Things will never get better for me," to which the nurse responds by asking, "Why are you so depressed?" Which statement describes the significance of this question asked by the nurse

It helps the nurse plan appropriate assistance for the client

The nurse is reviewing the medication history of a client with epilepsy who was recently diagnosed with bipolar disorder. Which medication would the nurse expect to be prescribed for both conditions

Lamotrigine

Which treatment modality is beneficial to a client who experiences winter "blues"?

Light therapy

Which physiological symptom is observed in a client with transient depression

Listlessness

The client hears the word "match." The client replies, "A match. I like matches. They are the light of the world. God will light the world. Let your light so shine." Which communication pattern does the nurse identify?

Loose association

The client hears the word "match." The client replies, "A match. I like matches. They are the light of the world. God will light the world. Let your light so shine." Which communication pattern does the nurse identify?

Loose association Rationale: Loose association is characterized by communication in which ideas shift from one unrelated topic to another. The situation in the question represents this communication pattern. Clang associations are groupings of words, usually rhyming words, that are based on similar-sounding sounds, even though the words themselves don't have any logical reason to be grouped together. Word salad is a "confused or unintelligible mixture of seemingly random words and phrases", most often used to describe a symptom of a neurological or mental disorder. Ideas of reference and delusions of reference describe the phenomenon of an individual's experiencing innocuous events or mere coincidences and believing they have strong personal significance. It is "the notion that everything one perceives in the world relates to one's own destiny."

The nurse should recognize which factors that distinguish personality disorders from psychosis?

Major disturbances of thought are absent in personality disorders.

A client with multiple sclerosis who is on steroid therapy is admitted to the psychiatric ward. Which symptoms does the nurse observe in the client

Mania

While caring for a client with psychotic disorder, the nurse infers that he is at risk of being violent with other individuals. Which symptom enabled the nurse to reach this conclusion

Manic excitement

A client is prescribed duloxetine. Which client education is most appropriate for the nurse to provide?

Medication should not be discontinued abruptly

After assessing a child, the nurse suspects the child has cyclothymic disorder. Which statement made by the child's parent supports the nurse's assumption

My child has periods of elevated mood from the age of 5 years. These episodes are more frequent since she turned 8 years old

Which statement made by a client with bipolar mania indicates personalizing thought

My sister is this happy only when she is with me

A paranoid client presents with bizarre behaviors, neologisms, and thought insertion. Which nursing action should be prioritized to maintain this client's safety?

Note escalating behaviors and intervene immediately Rationale: The nurse should note escalating behaviors and intervene immediately to maintain this client's safety. Early intervention may prevent an aggressive response and keep the client and others safe.

A client has been feeling "down in the dumps" because of the loss of a pet 2 years ago. Which condition would the nurse suspect in the client?

Persistent depressive disorder

A client tells the nurse at the methadone clinic that he has a prescription for methadone 40 mg daily orally. The nurse recognizes that methadone maintenance can be used as an effective drug replacement for individuals addicted to:

Opiates. Rationale: Medications like buprenophine (Subuxone) and Methadone have been employed in the treatment of morphine and heroin addiction

Which nursing intervention has priority as a patient diagnosed with anorexia nervosa begins to gain weight?

Observe for adverse effects of re-feeding

A client tells the nurse at the methadone clinic that he has a prescription for methadone 40 mg daily orally. The nurse recognizes that methadone maintenance can be used as an effective drug replacement for individuals addicted to:

Opiates

A patient is prescribed ziprasidone (Geodon) for the treatment of schizophrenia. Which of the following would alert the healthcare provider that the patient is experiencing an adverse effect of the medication? Select all that apply.

Palpitations and syncope Pulmonary crackles Increased temperature Rigidity and bradycardia (NOT seizure activity) Rationale:Medications rarely have just one action (such as decreasing symptoms of psychosis). Extrapyramidal symptoms (EPS) are common adverse effects of antipsychotics. Ziprasidone is associated with the development of agranulocytosis. Ziprasidone is associated with alterations in esophageal motility. In addition to blocking dopamine receptors, ziprasidone also blocks alpha-1 adrenergic receptors, which may result orthostatic hypotension. Orthostatic hypotension may also be caused by EKG changes. Alterations in esophageal motility increases the risk of aspiration. Agranulocytosis increases the patient's risk of infection.

Which of the following assessment findings in a patient's health history supports a diagnosis of substance dependence?

Patient experiences withdrawal symptoms when not using the substance. Rationale: These are characteristics of a wide spectrum of substance-related problems. Problems related to substance use tend to become more serious with repeated use leading to dependence. Substance dependence is characterized by the need to continually use the substance in order to avoid unpleasant physical symptoms of withdrawal (physical dependence), often accompanied by an intense craving for the substance (psychological dependence).

A patient is admitted to the medical unit after experiencing chest pain. Which of these additional findings would support a diagnosis of cocaine abuse?

Perforated nasal septum

A patient is admitted to the medical unit after experiencing chest pain. Which of these additional findings would support a diagnosis of cocaine abuse?

Perforated nasal septum. Rationale: Cocaine enhances monoamine neurotransmitter activity in the central and peripheral nervous systems. Cocaine reduces gastric motility and delays gastric emptying. If impaired liver function is present, it is usually related to viral hepatitis or concurrent alcohol use. Long-term intranasal use of cocaine is associated with a perforated nasal septum.

Which condition would the nurse suspect if a client who just gave birth complains of "feeling blue"?

Postpartum Depression

Which type of depression involves mood variation that is worse in the morning and gets better as the day progresses

Severe depression

A nurse recently admitted a client to an inpatient unit after a suicide attempt. A health-care provider orders amitriptyline (Elavil) for the client. Which intervention related to this medication should be initiated to maintain this client's safety upon discharge?

Provide a 1-week supply of Elavil with refills contingent on follow-up appointments. Rationale: The health-care provider should provide a 1-week supply of Elavil with refills contingent on follow-up appointments as an appropriate intervention to maintain the client's safety. Tricyclic antidepressants have a narrow therapeutic range and can be used in overdose to commit suicide. Distributing limited amounts of the medication decreases this potential.

When a client on an acute care psychiatric unit demonstrates behaviors and verbalizations indicating a lack of guilt feelings, which nursing intervention would help the client to meet desired outcomes?

Provide external limits on client behavior.

A client is diagnosed with bipolar II disorder with delirious symptoms of mania. Which intervention would the nurse include in the client's care plan

Provide information about support groups

Which nursing intervention would be most appropriate when caring for an acutely agitated client with paranoia?

Provide personal space to respect the client's boundaries. Rationale: The most appropriate nursing intervention is to provide personal space to respect the client's boundaries. Providing personal space may serve to reduce anxiety and thus reduce the client's risk for violence.

The nurse is caring for a client who is experiencing very frequent manic episodes. Which intervention by the nurse may help the client reduce the manic episodes of bipolar disorder

Providing a private room in a quit unit

Which actions by the nurse will help a hyperactive client achieve much-needed rest?

Providing a structured schedule of activities for the client

A client who is experiencing frequent manic episodes has a delusional thought process and low self-esteem. The nurse finds that the client is trying to manipulate others for self-gratification. Which interventions included in the client's care plan would the nurse expect to improve the client's self-esteem

Reinforcing acceptable behavior in the client

While caring for a postpartum client, the nurse suspects the client to have postpartum depression with psychotic features. Which symptom in the client made the nurse suspect this condition?

Rejection of the baby

A patient diagnosed with obsessive-compulsive disorder (OCD) continually carries a toothbrush, and will brush and floss up to fifty times each day. The healthcare provider understands that the patient's behavior is an attempt to accomplish which of the following?

Relieve anxiety Rationale: The patient diagnosed with OCD often recognizes the repeated actions are excessive and interfere with the patient's normal routine. The continual brushing and flossing are a result of persistent thoughts that compel the patient to perform the ritual. The ritualistic behavior (brushing and flossing the teeth) are compulsions which are performed in an attempt to provide relief from anxiety-provoking obsessions.

During a panic attack, a patient states, "I feel like I'm going to die!" The patient is hyperventilating, tachycardic, and reports feeling upper extremity numbness and tingling. Based on this patient's presentation, the healthcare provider would anticipate which additional clinical manifestation of the panic attack?

Respiratory alkalosis Rationale: An arterial blood gas will provide evidence of how the hyperventilation is affecting other body systems. Hyperventilation causes an increase in the volume of air the patient inhales and exhales. The patient is exhaling more carbon dioxide than normal, causing hypocapnia and respiratory alkalosis. The paresthesia reported by the patient is an additional clinical sign of alkalosis.

A patient is being observed for extrapyramidal symptoms. Which of these symptoms would alert the nurse to the possible onset of this condition? Select all that apply.

Restlessness Facial grimacing Tremors at rest (NOT blurred vision, flaccid extremities, or inability to concentrate) Rationale: The extrapyramidal system is a descending motor tract. The extrapyramidal system is involved in motor functions such as overall body coordination and posture. Extrapyramidal symptoms include motor restlessness (akathisia), tremors at rest (a symptom of parkinsonism), and facial grimacing (a symptom of dystonia).

A client is diagnosed with schizophrenia. A physician orders haloperidol (Haldol), 50 mg bid; benztropine (Cogentin), 1 mg prn; and zolpidem (Ambien), 10 mg HS. Which client behavior would warrant the nurse to administer benztropine?

Restlessness and muscle rigidity Rationale: The symptom of tactile hallucinations and reports of hearing disturbing voices would be addressed by an antipsychotic medication such as haloperidol. Tardive dyskinesia, a potentially irreversible condition, would warrant the discontinuation of an antipsychotic medication such as haloperidol. An anticholinergic medication such as benztropine would be used to treat the extrapyramidal symptoms of restlessness and muscle rigidity.

A patient is prescribed ziprasidone (Geodon) for the treatment of schizophrenia. Which of the following would alert the healthcare provider that the patient is experiencing an adverse effect of the medication? Select all that apply.

Rigidity and bradykinesia Palpitations and syncope Increased temperature Pulmonary crackle

Which personality characteristic is a nurse most likely to assess in a patient diagnosed with anorexia nervosa?

Rigidity, perfectionism

The nurse is providing teaching to a client being discharged on olanzapine. Which information would be important for the nurse to include in the teaching

Rise slowly from a sitting or lying position

A priority nursing diagnosis for a 25-year-old client admitted to a drug rehabilitation program 4 days ago for chronic cocaine abuse would be:

Risk for self-directed violence related to suicidal depression

A priority nursing diagnosis for a 25-year-old client admitted to a drug rehabilitation program 4 days ago for chronic cocaine abuse would be:

Risk for self-directed violence related to suicidal depression Rationale: One of the main symptoms of cocaine withdrawal is intense depression; therefore option 1 is a priority. Risk for noncompliance in option 2 is not the priority, and disturbed sensory perception is not the priority at this time. There is a need to sleep, so the hyperactivity mentioned in option 4 is incorrect.

A client diagnosed with bipolar disorder: depressive episode intentionally overdoses on sertraline (Zoloft). Family reports that the client has experienced anorexia, insomnia, and recent job loss. What should be the priority nursing diagnosis for this client?

Risk for suicide R/T hopelessnes

A client diagnosed with bipolar disorder: depressive episode intentionally overdoses on sertraline (Zoloft). Family reports that the client has experienced anorexia, insomnia, and recent job loss. What should be the priority nursing diagnosis for this client?

Risk for suicide R/T hopelessness Rationale: The priority nursing diagnosis for this client should be risk for suicide R/T hopelessness. The nurse should prioritize diagnoses on the basis of physical and safety needs. This client continues to be at risk for suicide related to an intentional Zoloft overdose.

A client diagnosed with brief psychotic disorder tells a nurse about voices telling him to kill the president. Which nursing diagnosis should the nurse prioritize for this client?

Risk for violence: directed toward others Rationale: This client is experiencing visual hallucinations. This nursing response is an example of "voicing or stating the obvious," which facilitates the patient to verbalize and validate the observation. This should be followed by presentation of reality and knowing the content of the hallucination.

Which nursing diagnosis should be prioritized when providing care to a client diagnosed with paranoid personality disorder?

Risk for violence: directed toward others R/T suspicious thoughts

Which medication would be beneficial to a client with autism

Risperidone

A client diagnosed with schizophrenia is slow to respond and appears to be listening to unseen others. Which medication should a nurse expect a physician to order to address this type of symptom?

Risperidone (Risperdal) to address the positive symptom Rationale: The nurse should expect the physician to order risperidone (Risperdal) to address the positive symptoms of schizophrenia. Risperidone (Risperdal) is an atypical antipsychotic used to reduce positive symptoms, including disturbances in content of thought (delusions), form of thought (neologisms), or sensory perception (hallucinations).

A client is diagnosed with anaclitic depression. Which could be the cause of such a condition?

Separation from mother during the first year of life

Which biochemical is estimated in the dexamethasone suppression test performed to determine if a client has somatically treatable depression?

Serum cortisol

Warren's college roommate actively resists going out with friends whenever they invite him. He says he can't stand to be around other people and confides to Warren "They wouldn't like me anyway." Which disorder is Warren's roommate likely suffering from?

Social anxiety disorder (social phobia) Rationale: Social anxiety disorder is an excessive fear of social situations R/T fear that one might do something embarrassing or be evaluated negatively by others.

When assessing a patient with severe depression, which of the following would the healthcare provider identify as a cognitive alteration?

Somatic delusions

When assessing a patient with severe depression, which of the following would the healthcare provider identify as a cognitive alteration?

Somatic delusions Rationale: Patients diagnosed with depression may experience cognitive, affective, behavioral, or physiological alterations. Cognition relates to processes such as judgment, evaluation, and reasoning. A somatic delusion, the false belief that the patient has some physical defect or disease (e.g. the patient might think he/she has an internal parasite), is a cognitive alteration associated with depression. The other choices are affective alterations.

A client who has been diagnosed with a phobic disorder asks the nurse if there are any medications that would be beneficial in treating phobic disorders. Which of the following would be accurate responses by the nurse? Select all that apply.

Some antidepressant agents have been successful in diminishing symptoms of agoraphobia and social anxiety disorder (social phobia). Beta-blockers have been used successfully to treat phobic responses to public performance. Specific phobias are generally not treated with medication unless accompanied by panic attacks. Some antianxiety agents have been successful in treating social phobias. Rationale: All of the listed pharmacological treatments are evidence-based treatments for phobic disorders.

Which category of drugs has the potential to initiate a manic episode? Select all that apply.

Sulfonamides Anitulcer agents Antihypertensives

Which cognitive symptoms are observed in a client with delirious mania? Select all that apply

Stupor Confusion Disorientation

A client is admitted with a diagnosis of brief psychotic disorder, with catatonic features. Which symptoms are associated with the catatonic specifier?

Stupor, muscle rigidity, and negativism

A client is admitted with a diagnosis of brief psychotic disorder, with catatonic features. Which symptoms are associated with the catatonic specifier?

Stupor, muscle rigidity, and negativism Rationale: Symptoms associated with the catatonic specifier include stupor and muscle rigidity or excessive, purposeless motor activity. Catatonic Behavior: Waxy flexibility or catalepsy Rigidity or posturing Negativism (resistance to a person moving them) Purposeless, unstimulated movement

Which term should a nurse use to describe the administration of a central nervous system (CNS) depressant during alcohol withdrawal?

Substitution therapy Rationale: A CNS depressant such as Ativan is used during alcohol withdrawal as substitution therapy to prevent life-threatening symptoms that occur because of the rebound reaction of the central nervous system.

For which condition would a black-box warning be issued for antidepressant use in young adults under 25, teens, and children

Suicidal thoughts

A nurse is discussing treatment options with a client whose life has been negatively impacted by claustrophobia. The nurse would expect which of the following behavioral therapies to be most commonly used in the treatment of phobias? Select all that apply.

Systemic desensitization Imploding (flooding) Rationale: The nurse should explain to the client that systematic desensitization and imploding are the most commonly used behavioral therapies in the treatment of phobias. Systematic desensitization involves the gradual exposure of the client to anxiety-provoking stimuli. Imploding is the intervention used in which the client is exposed to extremely frightening stimuli for prolonged periods of time.

A client is admitted to the psychiatric ward with a history of two manic episodes. On interaction, the nurse learns that the client has acute symptoms of mania. Which interventions would the nurse include in the acute phase treatment plan for this client? Select all that apply

Take measures to alleviate insomnia in the client Provide adequate quantiies of food and fluids Assist in performing personal hygiene and grooming Remove dangerous objects from the environment

A client diagnosed with schizophrenia takes an antipsychotic agent daily. Which assessment finding should a nurse immediately report to the client's attending psychiatrist?

Temperature of 104°F (40°C) Rationale: When assessing a client diagnosed with schizophrenia who takes an antipsychotic agent daily, the nurse should immediately address a temperature of 104°F (40°C). A temperature this high can be a symptom of the rare but life-threatening neuroleptic malignant syndrome.

The nurse is explaining the side effects of electroconvulsive therapy (ECT) to the guardian of a client with depression. Which relevant information would the nurse provide

The client exhibits confusion for short period of time

The nurse is caring for a client with complicated grieving. Which action of the client would be monitored to evaluate the effectiveness of the nursing care

The client expresses appropriate anger about the loss

A client diagnosed with alcohol use disorder joins a community 12-step program and states, "My life is unmanageable." How should the nurse interpret this client's statement?

The client has accomplished the first of 12 steps advocated by Alcoholics Anonymous. Rationale: The first step of the 12-step program advocated by Alcoholics Anonymous is that clients must admit powerlessness over alcohol and that their lives have become unmanageable.

A client is admitted into the psychiatric unit with symptoms of hypomania. The client's history indicates that the client has never experienced a full manic episode. Which statement is true regarding the nurse's suspicion regarding the client's condition

The client has bipolar II disorder

A client is diagnosed with cyclothymic disorder. What client behaviors should the nurse expect to assess?

The client has endured periods of elation and dysphoria lasting for more than 2 years. Rationale: The essential feature of cyclothymic disorder is a chronic mood disturbance of at least 2 years' duration, involving numerous episodes of hypomania and depressed mood of insufficient severity or duration to meet the criteria for bipolar I or II disorder.

A client diagnosed with schizophrenia tells a nurse, "The 'Shopatouliens' took my shoes out of my room last night." Which is an appropriate charting entry to describe this client's statement?

The client is expressing a neologism."

Which symptoms does the nurse expect in a client who shows hypomanic episodes of bipolar disorder

The client shows manic symtoms that last for atleast 4 consecutive days

A client is diagnosed with bipolar disorder and admitted to an inpatient psychiatric unit. Which is the priority outcome for this client?

The client will remain safe throughout hospitalization. Rationale: A client diagnosed with bipolar disorder is at risk for injury in either pole of this disorder. In the manic phase the client is hyperactive and can injure self inadvertently, and in the depressive phase the client can be at risk for suicide.

The nurse is caring for a client with major depressive disorder who verbalizes statements that reflect low self-esteem. The nurse tries to teach assertive techniques to the client. Which outcome can be expected in the client if the nursing intervention is successful

The client will try new activities without the fear of failure

A client who is diagnosed with major depressive disorder asks the nurse what causes depression. Which of these is the most accurate response?

The exact cause of depressive disorders is unknown. A number of things, including genetic, biochemical, and environmental influences, likely play a role.

A client who is diagnosed with major depressive disorder asks the nurse what causes depression. Which of these is the most accurate response?

The exact cause of depressive disorders is unknown. A number of things, including genetic, biochemical, and environmental influences, likely play a role. Rationale: Depression is likely an illness that has varied and multiple causative factors, but at present the exact cause of depressive disorders is not entirely understood.

A client who has undergone surgery for left ventricular dysfunction is receiving treatment for bipolar mania. Which medication may increase the risk of cardiac complications in the client

Verapamil

What is meant by the phrase off-label use of medications

The use of medication for a purpose other than what is approved by the FDA

Which characteristics should a nurse recognize as being exhibited by individuals diagnosed with any personality disorder?

These clients accept and are comfortable with their altered behaviors.

During an admission assessment, a nurse notes that a client diagnosed with schizophrenia has allergies to penicillin, prochlorperazine (Compazine), and bee stings. On the basis of this assessment data, which antipsychotic medication would be contraindicated?

Thioridazine (Mellaril), because of cross-sensitivity among phenothiazines Rationale: The nurse should know that thioridazine (Mellaril) would be contraindicated because of cross-sensitivity among phenothiazines. Prochlorperazine (Compazine) and thioridazine are both classified as phenothiazines.

A nursing instructor is teaching about the prevalence of bipolar disorder. Which student statement indicates that learning has occurred?

This disorder is more prevalent in the higher socioeconomic groups."

An adult client diagnosed with bipolar I disorder is prescribed lamotrigine (Lamictal), 400 mg three times a day, for mood stabilization. Which is a true statement about this medication order?

This dosage is more than twice the recommended dosage range.

An adult client diagnosed with bipolar I disorder is prescribed lamotrigine (Lamictal), 400 mg three times a day, for mood stabilization. Which is a true statement about this medication order?

This dosage is more than twice the recommended dosage range. Rationale: The recommended dose of lamotrigine for treatment of bipolar disorder in adult clients should not exceed 400 mg daily.

A nurse reviews the laboratory data of a 29-year-old client suspected of having major depressive disorder. Which laboratory value would potentially rule out this diagnosis?

Thyroid-stimulating hormone (TSH) level of 6.2 U/mL (normal for this age group is 0.4 to 4.2 U/mL) Rationale: According to the DSM-5, symptoms of major depressive disorder cannot be due to the direct physiological effects of a general medical condition (e.g., hypothyroidism). The diagnosis of major depressive disorder may be ruled out if the client's laboratory results indicate a high TSH level (normal range for this age group is 0.4 to 4.2 U/mL), which results from a low thyroid function, or hypothyroidism. In hypothyroidism metabolic processes are slowed, leading to depressive symptoms.

A nurse holds the hand of a client who is withdrawing from alcohol. What is the nurse's rationale for this intervention?

To assess for fine tremors Rationale: The nurse is most likely assessing the client for fine tremors secondary to alcohol withdrawal. Withdrawal from alcohol can also cause headache, insomnia, transient hallucinations, depression, irritability, anxiety, elevated blood pressure, sweating, tachycardia, malaise, coarse tremors, and seizure activity.

A client is diagnosed with dissociative identity disorder (DID). What is the primary goal of therapy for this client?

To collaborate among subpersonalities to improve functioning

A nurse begins the intake assessment of a client diagnosed with bipolar I disorder. The client shouts, "You can't do this to me. Do you know who I am?" Which is the priority nursing action in this situation?

To provide self and client with a safe environment

A nurse begins the intake assessment of a client diagnosed with bipolar I disorder. The client shouts, "You can't do this to me. Do you know who I am?" Which is the priority nursing action in this situation?

To provide self and client with a safe environment Rationale: During a manic episode the client's mood is elevated, expansive, and irritable. Providing a safe environment should be prioritized to protect the client and staff from potential injury.

A nurse admits an older client who is experiencing memory loss, confused thinking, and apathy. A psychiatrist suspects depression. What is the rationale for performing a mini-mental status exam?

To rule out neurocognitive disorder Rationale: A mini-mental status exam should be performed to rule out neurocognitive disorder. The elderly are often misdiagnosed with neurocognitive disorder such as Alzheimer's disease, when depression is their actual diagnosis. Memory loss, confused thinking, and apathy are common symptoms of depression in the elderly.

During the admission interview, a patient mentions to the nurse that he had increased the amount of oxycodone because the smaller dose "just wasn't doing it for me anymore." The nurse assesses this phenomenon as resulting from:

Tolerance. Rationale: Tolerance is a person's diminished response to a drug, which occurs when the drug is used repeatedly and the body adapts to the continued presence of the drug. Withdrawal are the collection of physical and mental symptoms that occur after stopping or reducing intake of a drug. Relapse of drug-seeking behavior, is a form of spontaneous recurrence of pathological drug use after a period of abstinence. Intoxication is the impaired condition caused by use of alcohol or a drug or other chemical substance.

Which type of depression occurs with everyday disappointments? Mild depression

Transient depression

Which symptoms can be seen upon the blockade of norepinephrine reuptake

Tremors Cardiac arrythmias sexual dysfunction hypertension

Anxiety disorders are the most common type of psychiatric disorders in the US.

True

Anxiety disorders have high rates of comorbidity with major depression and alcohol and drug abuse.

True

Beta-blockers such as atenolol, nadolol, or propranolol may be useful for treatment of situational/performance anxiety.

True

Caffeine-containing products, such as coffee, tea, and colas, should be discontinued (or decreased to a low reasonable level) for patient with anxiety disorders.

True

Escitalopram (Lexapro) is likely to cause fewer hepatic enzyme interactions and may be appropriate initial choices for patients with complicated medical regimens or those who are concerned about drug interactions.

True

In the central nervous system (CNS), the major mediators of the symptoms of anxiety disorders appear to be norepinephrine, serotonin, dopamine, and gamma-aminobutyric acid (GABA).

True

Patients with significant discomfort from acute anxiety can benefit from benzodiazepines.

True

Which suggestion is most appropriate for the nurse to give a client with transient depression

Try to focus on your goals and achievements to overcome depression

Which of the following defense mechanisms may be observed in a patient diagnosed with obsessive-compulsive disorder (OCD)?

Undoing Rationale: Defense mechanisms are used when there is a threat to the patient's psychological integrity. The patient diagnosed with OCD will use this defense mechanism to deal with intolerable levels of anxiety. The patient attempts to substitute the anxiety with a behavior which is maladaptive. Undoing is a way of symbolically canceling out (negating) an experience that the patient finds intolerable. The compulsive behavior is designed to counteract or undo the patient's obsession.

A parent has consulted the school nurse because she is concerned about her 16-year-old son and the possibility of drug abuse. What indication of substance abuse in adolescents should the nurse plan to teach the mother?

Unexplained moodiness

A parent has consulted the school nurse because she is concerned about her 16-year-old son and the possibility of drug abuse. What indication of substance abuse in adolescents should the nurse plan to teach the mother?

Unexplained moodiness Rationale: New friends (option 1) do not indicate a problem. With substance abuse, impaired academic performance may occur, not improved (option 2). Option 4 is incorrect as headaches do not necessarily indicate substance abuse. Moodiness (option 3) can be indicative of substance abuse.

A client diagnosed with bipolar I disorder: manic episode refuses to take lithium carbonate because he complains that it makes him feel sick. Which of the following medications might be alternatively prescribed for mood stabilization in bipolar disorders?

Valproic acid (Depakene) (Also Divalproex-Depakote) Rationale: Although lithium is a prototype drug in the treatment of bipolar disorders, anticonvulsants such as valproic acid also have demonstrated efficacy for mood stabilization.

The nurse reviews the prescription of a client with depression and informs the client, "You should make sure you slowly rise up from the sitting or lying-down position." What is the nurse expecting to control in the client

Variations in blood pressure

A client who has been on the unit for 2 weeks relates a 15-year history of polysubstance abuse with several detoxifications followed by relapses. The client mentions that when he is released from the hospital, he tries to "get on with my life and put this other stuff in the past." The nurse identifies an appropriate outcome as being that client will:

View recovery is a lifelong process occurring a day at a time

A client who has been on the unit for 2 weeks relates a 15-year history of polysubstance abuse with several detoxifications followed by relapses. The client mentions that when he is released from the hospital, he tries to "get on with my life and put this other stuff in the past." The nurse identifies an appropriate outcome as being that client will:

View recovery is a lifelong process occurring a day at a time Rationale: Options 2 and 3 are past issues and not as helpful as a future focus. Methadone maintenance is not appropriate for polysubstance abuse, so option 4 is incorrect. A helpful view of recovery is described in option 1.

A client is taking Clonazepam (Klonopin) for generalized anxiety disorder symptoms. In which situation should a nurse recognize that this client is at greatest risk for adverse effects from the medication?

When the client combines the drug with alcohol Rationale: Both Klonipin and alcohol are central nervous system depressants. In combination, these drugs have an additive effect and can suppress the respiratory system, leading to respiratory arrest and death.

If clozapine (Clozaril) therapy is being considered, the nurse should evaluate which laboratory test to establish a baseline for comparison in order to recognize a potentially life-threatening side effect?

White blood cell count Rationale: The nurse should establish a baseline white blood cell count to evaluate a potentially life-threatening side effect if clozapine (Clozaril) is being considered as a treatment option. Clozapine can have a serious side effect of agranulocytosis, in which a potentially fatal drop in white blood cells can occur.

Which side effects in the client who is on lithium carbonate therapy prompt the primary health-care provider to decrease the dose of the medication? Select all that apply.

blurred vision tremors

The nurse is caring for a client with symptoms of alcohol withdrawal. Which medication would the nurse prepare to administer

carbamazepine

The nurse is caring for a client with depression who reports burning and stabbing muscle pains due to nerve damage. The medical reports indicate that the client has a herpes zoster infection. Which medication would be most beneficial for the client

carbazaepine

The nurse is providing information about dietary restrictions to a client who was prescribed a monoamine oxidase inhibitor (MAOI) to treat depression. Which food product can the client consume safely while on MAOI therapy -beer -soy sauce -chocolate -cottage cheese

cottage cheese

Outpatient treatment is planned for a patient diagnosed with anorexia nervosa. Select the most important outcome related to the nursing diagnosis: Imbalanced nutrition: less than body requirements. Within 1 week, the patient will:

gain 1 to 2 pounds.

The most effective nursing approach to deal with denial in a client who abuses substances is

having the client identify the effects of substance use on his or her life.

The most effective nursing approach to deal with denial in a client who abuses substances is:

having the client identify the effects of substance use on his or her life. Rationale: It is most helpful for the client to develop an awareness that many life problems are related to substance abuse so option 3 is correct. Learning about the addictive personality (option 1) and the physiologic effects (option 4) are not as helpful with denial. To confront about a hopeless life situation (option 2) is not therapeutic.

A nursing diagnosis for a patient diagnosed with bulimia nervosa is: Ineffective coping, related to feelings of loneliness as evidenced by overeating to comfort self, followed by self-induced vomiting. The best outcome related to this diagnosis is, within 2 weeks the patient will:

identify two alternative methods of coping with loneliness.

The primary health-care provider suggests that the nurse schedule light therapy for a client early in the day. Which complication can be prevented in the client as a result of this intervention?

insomnia

Which interventions would the nurse implement while caring for a client with mania who is on lithium carbonate therapy? Select all that apply

instruct the client to keep appointments to have blood levels drawn notify healthcare provider if serum level reaches >1/5 mEq/L

A client gave birth to a stillborn baby and is diagnosed with psychosis. Which treatment would be most beneficial for this client

lithium carbonate

A client with bipolar disorder has been in an accident and has brain damage. Which medication is contraindicated in this client for the treatment of bipolar disorder

lithium carbonate

Which medication binds to the ANK3 (ankyrin G) protein to prevent manic episodes of bipolar disorder

lithium carbonate

While communicating with a client, the nurse suspects bipolar mania. The nurse also finds a decreased neutrophil count in the client's laboratory reports. Which medication would be most beneficial for this client

lithium carbonate

A nurse is caring for an adolescent client with major depressive disorder. Which medications will the nurse expect to be contraindicated in such clients? Select all that apply.

paroxetine duloxetine citalopram

Physical assessment of a patient diagnosed with bulimia nervosa often reveals

prominent parotid glands.

While providing health teaching for a patient diagnosed with bulimia nervosa, a nurse should emphasize information about

recognizing symptoms of hypokalemia.

A nurse suspects that the client is experiencing low self-esteem. Which objective data would support the nurse's suspicion

the client does not make eye contact

While interacting with a client, the nurse suspects that the client is having a manic episode of bipolar disorder. Which statement supports the nurse's suspicion

the client has elevated or irritable mood

During the admission interview, a patient mentions to the nurse that he had increased the amount of oxycodone because the smaller dose "just wasn't doing it for me anymore." The nurse assessess this phenomenon as resulting from

tolerance


Related study sets

Chapter 33: The Great War: The World in Upheaval

View Set

Complete First - Unit 5, phrasal verbs pgs. 53-54

View Set

absolutism/ Scientific Revolution/ Enlightenment quiz

View Set

Salesforce Marketing Cloud Administrator - Practice Questions

View Set

ACCT 212 Chapter 6: Cost-Volume-Profit Relationships

View Set

Fluid, Electrolyte, and Acid-Base Balance Prep U Questions- Foundations

View Set

Chapter 13 - The Radiate Animals: Phylum Cnidaria, Phylum Ctenophora

View Set